2016 AMC 12A Problems/Problem 3

Revision as of 23:14, 3 February 2016 by FractalMathHistory (talk | contribs) (Created page with "==Solution== <cmath>\text{rem}\left(\frac{3}{8},-\frac{2}{5}\right)</cmath> <cmath>=\frac{3}{8}-\left(-\frac{2}{5}\right)\lfloor\frac{\frac{3}{8}}{-\frac{-2}{5}}\rfloor</cmath...")
(diff) ← Older revision | Latest revision (diff) | Newer revision → (diff)

Solution

\[\text{rem}\left(\frac{3}{8},-\frac{2}{5}\right)\] \[=\frac{3}{8}-\left(-\frac{2}{5}\right)\lfloor\frac{\frac{3}{8}}{-\frac{-2}{5}}\rfloor\] \[=\frac{3}{8}+\left(\frac{2}{5}\right)\lfloor -\frac{15}{16}\rfloor\] \[=\frac{3}{8}+\left(\frac{2}{5}\right)\left(-1\right)\] \[=\frac{3}{8}-\frac{2}{5}\] \[=\boxed{\textbf{(B)}-\frac{1}{40}}\]